false
Catalog
Multiprofessional Critical Care Review: Adult 2024 ...
Session 1 Recording
Session 1 Recording
Back to course
[Please upgrade your browser to play this video content]
Video Transcription
Hello and welcome to today's Q&A session on the topics of respiratory cardiovascular disease hemodynamics and nephrology. I'm Dr. Pastores from Memorial Sloan Kettering Cancer Center. Welcome to the session. This webcast is being recorded. The recording will be available in 24 to 48 hours in the content section of the course. Please note the disclaimer stating that the content to follow is for information or educational purposes only. This session will be highly interactive with polling questions and the ability to ask questions of the faculty. When a polling question pops up, simply select your answer and click submit. If you have a question for the faculty, please use the Q&A section and click submit. Use the toolbar for the Q&A. If you would like to chat with other attendees, please use the chat area. Today's faculty consists of Dr. Skalila Gates from Northwestern University in Chicago, Michael Connor from Emory University, Sami Zakaria from Johns Hopkins, Phil Efron from University of South Florida, and Stephen Hallenberg from Hackensack Meridian in New Jersey. None of the faculty today have anything to disclose. Without any further ado, Dr. Hallenberg, please get started with your first question. Thank you. It's a pleasure to be here. Not quite as much a pleasure as it would have been if we were in Chicago, but so be it. I would like to say that like the rest of the panelists, we didn't write these questions. We're just explaining them. You might have some objections to the questions and I might actually share some of them. You can actually put that in the Q&A. That said, the test itself isn't full of perfect questions. You might actually have to answer an imperfect question. Let's start the first one. My understanding is that you'll have a chance to, you can start responding at any time. A 56-year-old man is admitted to the ICU with hypoxemic respiratory failure with an oxygen saturation of 76%. And he requires intubation. His wife says he has heartburn-type syndrome symptoms for the past three days and became increasingly breathless for an hour before admission. On physical examination, he has diffuse arousal, elevated jugular venous pressure, and a short early systolic murmur heard at the apex. Extremities are cool. EKG demonstrates sinus tachycardia with Q waves in the infralateral leads and nonspecific ST segment changes. Chest radiograph demonstrates diffuse bilateral infiltrates. Left ventricular function is hyperdynamic with left ventricular ejection fraction 70% and focal akinesis of the mid and distal infralateral and inferior segments. Valvular structures are not well visualized. Which of the following is the most appropriate next diagnostic test? CT angiography of the chest, transesophageal echocardiography, bronchoscopy with BAL, serum procalcitonin level, and cardiac nuclear perfusion scan with regadenosine. That would be an adenosine preparation. All right. Am I supposed to be seeing responses? There we go. Well, we got one for one. Okay. So, yes, transesophageal echocardiography, we can go to the next slide. But they're essentially giving you a patient with pulmonary edema. And in the context of an MI, they're looking for a complication of MI. And this would be papillary muscle rupture. And you sort of have a short systolic ejection murmur because the pressure is equalized and focal akinesis and they tell you that. And they want an echocardiogram to diagnose this. Of the available answers, transesophageal echocardiography is, in fact, the best answer. In fact, if you know about Doppler, you can find the MR on a transthoracic echo. But of the available answers, this is clearly the best answer. And, again, complications of myocardial infarction are actually getting less common with early revascularization. But complications of myocardial infarction are overrepresented on the exam and are not getting less common on the exam. So you should watch out. Papillary muscle rupture, free wall rupture, VSD, right ventricular infarction, they're very common on the exam. And you should sort of familiarize yourself with them before the test. Okay. There's a reference here on the next slide, I think. Which is a reasonable reference. Let's go on to the next question. A 75-year-old man presents to the emergency department with acute chest pain that radiates to the back. Initial blood pressure is 160 over 90. Yeah, when this poll comes up, it blocks my slide. And heart rate is 100 beats a minute. EKG and troponin are negative. Chest CT is negative for pulmonary embolus, but there is evidence of an intimal flap in the descending thoracic aorta, which is the most of the following is the most appropriate initial management. Open surgical repair, endovascular repair, medical management to achieve heart rate less than 60 and systolic blood pressure 100 to 120, or medical management to achieve heart rate less than 100 beats per minute and systolic 120 to 140. All right, I got two for two, with the correct answer here. So yes, this one is a descending aortic dissection. And the key with descending aortic dissections is that you want to decrease shear stress on the myocardium that involves decreasing both contractility and blood pressure. And so usually you use a beta blocker along with a vasodilator. You don't want to use a vasodilator alone and lower is better when it comes to the systolic blood pressure. You get in trouble if you start doing these. These are in descending aortic dissections are almost always in people, older people with lots of comorbidities and chronic hypertension, and often diabetes and you'd really like not to touch these, if you want, if you can. So the first therapy is medical therapy only if you wind up with complications would you proceed to a surgical option. We can go on to the to the references here. There actually are newer guidelines that aren't referred here but these references are are reasonable. I believe the newer guidelines are 2020 but don't quote me on that. Okay, third question. 55 year old patient was admitted to the ICU with dyspnea on exertion orthopnea for the past two weeks. Vital signs are temperature 36 nine heart rate, 82 respiratory rate 14 and oxygen saturation is measured by pulse oximetry is 93% on room air. The patient is awake and following commands. Physical examination reveals a pansystolic murmur louded in the fourth in loudest in the fourth intercostal space in the parasternal region. Lung sounds are clear. Patient also has two plus edema serum electrolytes reveals essentially nothing with a low ionized calcium triple lumen catheter is inserted to obtain right atrial pressure, because they don't want to do PA catheters anymore EKG and right atrial tracing are shown below, which of the following is the most likely diagnosis, tricuspids, and we'll go to the slide. Let's go to the next slide we'll show you the picture. So the choices are on the next slide there's a picture, there we go. The choices are tricuspid stenosis, tricuspid regurgitation, mitral stenosis and mitral regurgitation. So we should get the panel to vote, we need some numbers. All right. Um, so that actually, that actually is all the panel can vote Oh well. Um, that actually is a reasonable answer based on the clinical scenario, but is not in fact the answer that they're looking for and I actually think this is a good. Um, this is a good way to get some testing. Um, so, uh, so let's let's go to the tracing first let's go back and they're having you read the tracing so remember this is a right atrial tracing and they're showing you a big V wave on the right atrial tracing. And because of that, you probably ought to answer tricuspid regurgitation, not that the patient probably doesn't have mitral regurgitation, but they definitely have tricuspid regurgitation on the basis of this tracing so just take the easy answer, even though, even though on clinical grounds, it may not be the right answer. So, and they'll do this to you on the test just look at what they're giving you, they're showing you the right side, take the right side and answer you'll get the question right. Okay. Next. Oh, there's a reference somewhere. Okay. Patient with acute myocardial infarction has received IV beta blockers and aspirin. He has a blood pressure of 85 over 60 and the pulse oximeter is no longer tracking well. Chances for a long term favorable outcome will be best with which of the following interventions rapid thrombolysis intraartic balloon pump therapy rapid cardiac catheterization, or immediate volume resuscitation. All right. Let's see what we got. I'm not seeing answers, but. 100% of the people have selected the rapid cardiac catheterization. Good. Excellent. Excellent. All right. So, that is the correct answer. So, again, look at what they're giving you. They're giving you a blood pressure of 85. They're giving you an acute MI with a low blood pressure, and the pulse oximeter isn't tracking well. That you really are supposed to look at, right? They're telling you that for a reason, and the reason is that there's no pulse pressure. So, this is cardiogenic shock, and thrombolysis doesn't work in cardiogenic shock. The IABP shock two trial actually shows that intra-aortic balloon pumps don't change outcome here. So, you want to go to the cath lab. You want to open up the artery. Volume resuscitation, again, not in this scenario. You should suspect pulmonary edema, even though they don't give you a lung exam. And so, you need to go to the cath lab rapidly. Whether you put in a mechanical support device is an issue for another day and not a choice in this question. All right. So, let's go to the references for a second. The guidelines, the guidelines, this is actually the latest version of the guidelines. Probably should update those, and that's a meta-analysis looking at showing that PTCA is probably better than thrombolytic therapy, but the cardiogenic shock trial, which is not on this trial, is Judy Hochman's trial. All right. Next question. 77-year-old Caucasian American man, is Caucasian American, is admitted to the ICU in the late afternoon for observation after an uneventful revascularization of his right lower extremity. With a bypass of his fempop, he has a past history of diabetes and a history of cardiac disease with a bypass craft 10 years ago. He has required intermittent libidolol and nitroprusside to maintain his systolic blood pressure less than 160. In the early evening, the patient is noted to become more agitated and is thought to be sundowning. Throughout the next several hours, the on-call resident gives the patient 30 milligrams of IV haloperidol. The nursing supervisor calls you at 2 a.m. and explains that your patient has just been resuscitated from a chaotic ventricular rhythm. Review of the patient's admission EKG reveals evidence of a previous myocardial infarction and the prolonged QTC. The most likely reason for the arrest is cyanide toxicity, diabetic cardiomyopathy, beta blockade in a patient with autonomic insufficiency or haloperidol. So we'll go on without much. So this in this question, so this answer is haloperidol. So they're pretty much giving this to you here. They've given you, so remember that they're haven't shown you the torsad because I guess because they figure that's too easy. Um but uh and and usually when they call you you from resuscitated from rhythm you kind of look at the rhythm but in any case they gave you the torsad. Remember that torsad de pointe is a clinical syndrome that by definition requires a long QT and polymorphic VT. Polymorphic VT without a long QT is in fact an ischemic rhythm but um they're giving you polymorphic feeds or they're giving you chaotic rhythm with a prolonged QT and you're and this is a pharmacology question. You're supposed to know that haloperidol uh otherwise known as haldol um lengthens the QT interval uh 30 milligrams is a big dose particularly for a 77 year old guy. Um so uh but you should you should look at uh particularly uh the torsad questions are a little easy a little too easy when they actually show you the rhythm just presumably why they went there but you should think about long QT uh medications that lengthen the QT and torsad. All right. Next question a there's a little bit of torsad references. A 61 year old man was recently hospitalized for treatment of COPD developed acute severe chest pain and dyspnea upon arrival in the emergency room at EKG is obtained in figure A and let's go to figure A. I'll keep reading for you. Um here's figure A. Uh the patient is pain-free after treatment with aspirin, clopidogrel, IV heparin, and IV nitroglycerin. Uh I'm going to show you another EKG but I'll let you look at this one for a couple of seconds and I'll read the choices based on the EKGs which are the following diagnoses is most likely. Core pulmonary with decompensation in this patient they told you about acute PE, acute severe chest pain and dyspnea, acute pulmonary embolus, acute pericarditis, presumably an EQAG diagnosis, acute coronary syndrome due to severe stenosis of the right coronary artery, or acute coronary syndrome due to severe stenosis of the left main coronary artery. This is not such a great question because two of those answers look pretty similar. They're either both wrong or one of them is right. Uh we'll go to the next slide. This is the one this is the second EKG when they're pain-free. Um and so the question is what do we think this is? And um I don't know whether I'm going to see answers but um so this is an EKG question um and this is what your the ST segment uh depression is pretty obvious. It's in fact anterolateral but what they really want you to look at is the ST elevation in lead AVR. If you go to the next slide that first EKG um and uh I don't know I I don't know whether you can see does my arrow showing up probably not. Um but in there in lead AVR is dramatic uh elevation and lead and and lead AVR with ST depression. Dramatic ST depression pretty much everywhere. If you go to the next one uh the ST depression is a little bit better but the elevation in AVR. So elevation AVR goes with left main coronary arteries disease that's actually a good tip. Um this guy despite the fact that he's uh that he's looking I guess sort of looking okayish clinically still should be suspected to have um to have a ST elevation MI in the anterolateral leads it needs to go to the cath lab. Um the really the other the other diagnoses are at least plausible in the clinical scenario but not with this EKG. Pulmonary emboli doesn't look EKG doesn't look like that. This is not pericarditis you're supposed to read the ST segments. All right next question. A 65 year old woman admitted for aneurysmal subarachnoid hemorrhage presents with dyspnea and substernal chest pain two days after admission. EKG shows anterior lead ST elevations and troponin level is mildly elevated. However there is no angiographic evidence of obstructive coronary artery disease or acute plaque rupture. She then developed hypotension to a blood pressure of 60 over 80 and is found to have moderate left ventricular outflow tract obstruction. Which of the following is contraindicated in treatments of this patient? Alpha agonists, beta blockers, inotropic agents, or intra-aortic balloon pumping. So they're kind of giving you this diagnosis. This is this is at least based on the EKG and the troponin. This is likely to be stress cardiomyopathy in the setting of an aneurysmal subarachnoid hemorrhage. Those used to be called cerebral T waves and I and most other people now believes there's no such thing. All of this is stress cardiomyopathy. And so she develops they give you moderate left ventricular outflow tract obstruction and if that's your problem alpha agonist to raise the blood pressure a good idea. Beta blockers are sort of okay. Maybe that decreases left ventricular contractility and helps with the obstruction. Don't really give you any EKG or left ventricular function. Inotropic agent is probably not a good idea because you're going to make the outflow tract obstruction worse. And intra-aortic balloon pumping again probably supports the heart not an unreasonable answer. So if they give you the outflow tract obstruction this is the right answer. Now if we go on to the here here we have a rationale says that the obstruction is seen in 13 to 18 patients. Most of this literature comes from the Mayo Clinic. In the Mayo Clinic world they see this in 20 percent of cases. I don't know about you in my world I see it almost never. I don't know that I really believe that 20 percent. That said in this test question when they give you the outflow tract obstruction without giving you echo findings then this is a question about outflow tract obstruction not what percentage of time does it occur in stress cardiomyopathy. And so the answer is still correct even if you don't believe the epidemiology of it. All right I'm done with my questions and I'm going to hand this one off to my colleague from Hopkins Sammy Zachariah who's going to take it from here. Hi good morning thanks Steve. So this is a 75 year old man who's admitted to the ICU with shortness of breath lightheadedness and bradycardia. His ECG is shown in the next slide we'll show that. Which of the following is the most appropriate next step in his management? So just to go over the answers while you're looking at this slide you could go back to this one. So since this is a benign rhythm no treatment is needed. Answer B is to administer atropine. Answer C is to administer IV amiodarone and answer D is to apply transcutaneous pacing paths. So since the polling is not working let's just go to the next slide and tell you the answer. So the answer is D which is that you want to apply transcutaneous pacing path. So if you go back to the ECG slide. So notice the pattern here. Here you have periods of group beating which is especially prominent in the latter portion of the strip but every RR interval is the same except when you have a drop beat. So whenever you see group beatings this patient has to have some type of second degree heart block and what you're trying to do is differentiating between type one block from the which is really benign and it's due to increased vagal node in the vagal tone in the AV node from type two block which is basically due to degeneration of the electrical conduction system and it leads to complete heart block. So this patient in particular has a MOBIS 2 second degree AV block. MOBIS 2 block it always results from a conduction system disease below the atrioventricular node. Medications of myocardial ischemia may cause such abnormal conduction and should be investigated but the immediate concern is for the progression of the conduction defect leading to a more serious hemodynamic issue such as complete heart block. Atropine doesn't help this at all and neither would bronchodilator therapy or other inotropes and you really what you need to do is to pace the patient while you perform further evaluation and to stabilize them. So let's go to the next slide. Let's go to the next slide after that. So this is just a reference from it. This is the guidelines for device-based therapy but there's newer ones that came out in 2018. So let's go to the next question. So during the past several months after inferior wall myocardial infarction a 42-year-old woman reports a frequent fluttering sensation associated with near syncope. She was discharged from the hospital after her myocardial infarction with quinidine for the treatment of paroxysmal atrial fibrillation. Just an editorial note we rarely use quinidine but you should be cued in that there's something maybe related to this medication. While in the hospital for evaluation she suddenly becomes dizzy and reports an irregular heartbeat. The tracing shown is obtained. Which of the following is a therapeutic agent of choice for dysrhythmia? And so the options are procainamide, magnesium, disipiramide, and potassium. So let's go to the next slide. Give you a few seconds before we decide. So to me this looks like polymorphic ventricular tachycardia and it kind of has a twisting pattern so it's really torsades to the flaw. So let's just go to the rationale. So the answer to this is whenever you see whenever you see this type of rhythm you see a polymorphic polymorphous ventricular tachycardia characterized by twisting of the ventricular complexes you want to be keyed into a few things. Number one it could be due to myocardial ischemia but most likely it's due to some type of medication effect. Most likely it's due to prolonged QT interval and the causes of torsades include drugs, myocardial ischemia, or infarction. Myocarditis could also do this, central nervous system disorders, congenital long QT syndromes, and a lot of electrolytes. Most commonly though in the ICU setting it's due to anti-rhythmic agents or some other medication. This patient was on quinidine which is a class 1a or anti-rhythmic and it prolongs the QT interval. Other ones in that class include procainamide and disipiramide. The other class that does this is class 3 agents such as amiodarone and sotalol. This patient got quinidine which is the most frequently reported drug associated with torsades and it could cause this quinidine syncope syndrome especially when the drug is at toxic levels. The best treatment for torsades is magnesium and a lot of it. You really need to give at least one to two grams given over several minutes and if that doesn't work you could give it again several times. If magnesium is ineffective you could overdrive the pace, the atria, and the ventricles to prevent further recurrences and what you're doing when you're doing that is preventing PVCs from forming and causing polymorphic VT. So the correct answer is magnesium. The other choices don't make sense and there's also no role for potassium in the treatment of torsades. Next slide. So this is a 20-year-old man who has a diagnosis of dilated cardiomyopathy and is admitted to the ICU. Congestive heart failure refractory to medical management necessitates a placement of a biventricular assist device. In the immediate post-operative period the left ventricular assist device is set at a rate of 70 and the driving systolic and diastolic pressures are 180 and 45 millimeters of mercury respectively. The right ventricular assist device is set at a rate of 70, so similar to the left one, and the driving systolic and diastolic pressures are 160 over 35. The intensivist is called to the bedside as the patient has suddenly developed an abnormal cardiac rhythm as shown. He's awakened following commands. Which of the following is the most appropriate immediate course of action for managing the abnormal rhythm? So let's go to the next slide and I'll go over the answers while we're looking at the slide. So the answers are immediate defibrillation with a dose of 200 joules, starting chest compressions, amniota on 300 milligrams, magnesium 2 grams, or no intervention. So just to show you here, in this case patient you can look at the ECG straight tracing and then it shows basically ventricular fibrillation. However, look at the blood pressure and SpO2 tracings which show normal essentially contractions and normal blood pressure. This patient has a blood pressure and he's mentating. So the dominant contributor to cardiac output in this patient is the biventricular assist devices. In patients with the biventricular assist devices, both the ventricles are well supported with the devices. So the presence of ventricular fibrillation has no impact on effective cardiac output. The only time such patients would need to be converted to more stable rhythm urgently or emergently is when they require an assist device exchange. You know, this is a little bit different in patients who only have a left ventricular assist device because those patients, ventricular fibrillation may affect right ventricular cardiac output and so it requires immediate treatment. But even in those cases, patients don't generally have a loss of blood pressure and generally they don't feel well. So you have time to sedate the patient and perform cardioversion. So the correct answer for this patient is to do nothing immediately. So let's go to the reference list. So here's a good reference. It's from 1994, but it hasn't really changed. The bottom line is patients with biventricular assist devices, if they develop ventricular or atrial arrhythmia, you know, you don't have to treat them emergently because those patients generally are stable in the short term. All right, let's go to the next slide. So the interaortic balloon pump pressure tracing shown in the next slide in figure A is most likely associated with, why don't we go to the next slide, here we go. So here's the answers, correct timing of balloon inflation, early balloon inflation, late balloon inflation, early balloon deflation or late balloon deflation. So take a look again at the tracing. So the answer for this tracing is that there's early balloon inflation. So what you want is you want the balloon to inflate right after the dicrotic notch. If it inflates early, then what you're doing is basically increasing afterload during ventricular systole, which is not ideal. So you would want to adjust the balloon pump to pump a little bit later, to inflate a little bit later. So let's go to the next slide to kind of show the tracing here. So the first tracing is unassisted systole, and notice how it goes up smoothly and then goes down, and then you'll see a dicrotic notch. That's when the aortic valve is closing. Then the patient will have unassisted aortic and diastolic pressure, and then you see the second beat is unassisted systole again. Then as the blood pressure drops, you'll see that the balloon is now inflating at where the dicrotic notch should occur. That is the perfect timing for balloon pump inflation. The blood pressure should increase at that point, so you'll have diastolic augmentation. So this increases coronary perfusion because you're driving blood into the corner of your arteries. Then the balloon deflates, and then your end diastolic pressure is actually lower, and then the assisted systole, the next beat would be lower as well. So the correct answer is early balloon inflation in this patient's case. So just to reiterate, the next slide just kind of discusses the rationale behind this. Just to repeat, balloon inflation should always occur at the beginning of diastole for optimal hemodynamic benefit. Diastole begins at the aortic valve closure, which is represented on the arterial pressure tracing as the dicrotic notch. Balloon inflation should occur at this point, and it will provide maximal augmentation of the diastolic blood pressure. If it occurs too early, the dicrotic notch will not be visualized, and the balloon augmentation waveform will be superimposed upon the left ventricular systolic component of the aortic pressure waveform. At that point, inflation will also start with the aortic valve still open, which is not good. Balloon deflation should occur at the end of diastole. It's represented in the arterial pressure tracing as the beginning of the systolic pressure waves. If this happens, you can have premature closure of the aortic valve, loss of the slow ejection phase of the left ventricle, decreased stroke volume, increased left ventricular and diastolic volume and pressures, and increased wedge pressures, more stress, aortic regurgitation, and increased myocardial injection consumption and work. So basically, all that's bad. So you'd want to adjust the tracing to make it lower. So I think that's my last question. I want to turn it over to Dr. Parula Gates, who can answer the next one. All righty. Thank you, Dr. Zachariah. So now we're going to move on to a little bit of pulmonary respiratory ICU. Our first case is in which of the following scenarios is non-invasive ventilation after extubation most appropriate. A is a 50-year-old man after a cabbage with rapid shallow breathing index of 60 on post-op day number one. The second is a 62-year-old man after esophagectomy who required re-intubation on post-op day one. The third is 68-year-old woman intubated three days ago for an exacerbation of COPD. And the last question D, or last option D, is 74-year-old man with community-acquired pneumonia who was extubated yesterday and now has worsening respiratory failure. Because our polling isn't working, I'll just sit here at the question, and we'll walk through what is right and what's wrong with the other options. And so the correct answer here is actually C. It is the 68-year-old woman intubated three days ago for COPD exacerbation. And so that is one of the indications for extubation to non-invasive ventilation is patients in which they are deemed to be high risk for re-intubation. And those are patients with respiratory failure, age over 65, so and obesity. So high risk for re-intubation. A is incorrect in that this gentleman post-CABG is doing fine, has a wonderful RSVI, and so should do well. Although more recent data suggests that perhaps he would benefit at least from extubation to high flow nasal cannula, but not non-invasive ventilation. B is incorrect as we do not use non-invasive ventilation or positive pressure ventilation in patients who are post-aposophagectomy. And D, we have a gentleman who has unfortunately failed extubation, so simply needs to be re-intubated. And so we can go to the next slide. And so that is the rationale, and we can go to the references. So we have three references. We'll have to update this to reflect the most recent data suggesting use of high flow nasal cannula in a population of people as well. So next question. The next question, we have a 78-year-old man with end-stage chronic obstructive pulmonary disease who comes to the emergency department with acute on chronic dyspnea, hypoxia, and hypercarbia. After an unsuccessful trial of non-invasive ventilation, he is intubated for respiratory failure. His initial ventilatory settings are assist control volume cycle with a tidal volume of 500, a respiratory rate of 20 breaths per minute. His FiO2 is at 100%, and he has a PEEP of 5. He is sedated on propofol to enrichment agitation sedation scale score of minus 2. His peak airway pressures are 50, and his plateau pressures are 30. But bedside ultrasound shows bilateral lung sliding. On auscultation, he has diffused bilateral wheezes. So which of the following is the most likely cause of his elevated airway pressures? Do we have intrinsic PEEP, also known as auto PEEP? Do we have a pneumothorax? Do we have endotracheal tube obstruction with secretions, or do we have a bronchospasm? And so the correct answer here is actually A, intrinsic positive in-expiratory pressure, also auto PEEP. So we can walk through the, we can stay at the question. We can walk through the stem of the question, and we know that it's not a pneumothorax as we have bedside ultrasound that actually shows bilateral lung sliding, suggesting that there is no pneumothorax there. Endotracheal tube obstruction as well as bronchospasm are potentially correct answers in that you have elevated peak airway pressures, which would then translate into increased resistive pressures. But what you would not explain is the elevated plateau pressures. And so the only thing that would give you increased plateau pressures, in addition to increasing your peak airway pressures, would actually be auto PEEP, which is something that we absolutely have to adjust the ventilator settings to fix, which would be prolonging the expiratory time to avoid any additional trauma and potentially pneumothorax at that point. We can go to the next question. All right, so the next question, we have a 36-year-old woman with cervical cancer who is admitted to the ICU and septic shot. She is adequately resuscitated, but on day two she develops hypoxemia and requires intubation. She is subsequently diagnosed with acute respiratory distress syndrome. The next day she continues to have hypoxemia despite treatment with the high FiO2 and low tidal volume ventilation. Which of the following steps should be taken to reduce her risk of mortality? So choice A would be airway pressure release ventilation. B would be adding inhaled nitric oxide. C would be high frequency oscillatory ventilation. D, proning positioning. Or E, extracorporeal membrane oxygenation. So the correct question or the correct answer here is actually D, prone positioning. It is the one of these options, the only of these options that actually has been shown to improve mortality based on the PERCEIVA trial, which indicated that patients with a persistent P to F ratio of less than 150 should be considered for proning with improved mortality. So we can go to the next question. And there's our PERCEIVA reference. So the next question, we have an 82-year-old woman who presents to the emergency department from home with altered mental status, productive cough, and shortness of breath. Her vital signs include a pulse of 115 beats per minute. Her respiratory rate is 40 breaths per minute. The blood pressure is 90 over 50, and the oxygen saturation is 78% of room air. The patient is awake and alert, but is confused. On examination, she is tachypneic and has diffused ronchi. The patient is placed on a non-rebreather mask, and oxygen saturation improves to 94%. Portable chest radiography shows multifocal areas of consolidation. Her white count is elevated 19,000 with 10% bands. Her arterial blood gas is there. We can see 7.35, 35, 65, with a bicarb of 18. And she's saturating 90% on 100% non-rebreather. The patient is transferred to the ICU. So the question is, what is the next best option in managing the patient's acute hypoxic respiratory failure? Your first choice is to continue the patient on the non-rebreather mask since her oxygenation is sufficient. Choice B is start patient on humidified high flow oxygen. Choice C is to start non-invasive positive pressure ventilation. And choice D is to intubate the patient and place her on mechanical ventilation. I'll give you just a sec to think about it. And the correct answer here is actually, remember it says the best next option, and it is to start the patient on humidified high flow oxygen. Humidified high flow oxygen does show reduction in mortality, and it reduces the need for mechanical ventilation, thus reducing the morbidity associated with mechanical ventilation. When we use high flow oxygen or high flow nasal cannula, we can theoretically provide some level of PEEP that may help improve oxygenation. We can provide some mucus clearing devices that can then help with this, what appears to be multifocal pneumonia as well. Additionally, high flow nasal cannula reduces the space and so can reduce the CO2 as well. And so although high non-invasive ventilation and mechanical ventilation are potential options for this, the best to prevent any additional morbidity would be to at least try the patient on humidified high flow oxygen. We can go to the next question. All right, so here we have a 30-year-old man who is placed on peripheral VA ECMO using femoral artery and vein catheters for worsening cardiogenic shock due to viral myocarditis, using femoral artery and vein catheters for worsening cardiogenic shock. During the following week, his native cardiac function appears to be improving, although he develops ventilator associated pneumonia and is being treated with broad spectrum antibiotics. His arterial blood gas drawn from his right radial artery shows a pH of 7.48, a partial arterial carbon dioxide pressure of 32. His PAO2 on that right radial arterial line is 60. The oxygen saturation is 90%, and that is on 60% FiO2 and 7.5 of P. The tidal volume is 450. His respiratory rate is 18. His chest x-ray shows a right middle lobe and left lower lobe infiltrate. His post-oxygenator ABG readings are a pH of 7.40, a PACO2 of 40, a PAO2 of 300. The nurse alerts the attending physician to the patient's declining oxygen saturations throughout the day and the low partial arterial oxygen pressure. Which of the following in this case is the most appropriate next step? So you could either change out the oxygenator, you could add a second oxygenator to the circuit, you can consider changing to central venal arterial ECMO or increasing circuit flow, you could increase the ventilator FiO2, or you could convert to venal venous ECMO. So the correct answer here would actually be C, to consider changing to to consider changing to central VA ECMO or increasing circuit flow. We can go to the rationale for this one. So what this is showing us is a potential complication of VA ECMO called the North-South syndrome. And what we see is that the right radial oxygenation was lower than the oxygenator. And so that suggested that there was not enough arterial oxygen supply to the upper portion of his body. If there's adequate native cardiac function with concomitant respiratory failure, the mixing of the retrograde blood flow from the femoral artery can be more distal and that leads to this upper body hypoxemia. We obviously monitor it by the way that we did here taking ABGs from the right radial, the upper limbs as well as looking at the post-oxygenator blood. And so how do we fix it? More importantly, the solution to this problem is basically to change the central cannulation. We were peripheral at the beginning, or we could increase the ECMO flows. And so that is the option. There was another option of considering changing to veno-venous ECMO if the cardiac function was adequate. And though it does say that it was improving, we don't know if it's adequate. So that wasn't really an option at that time. And then increasing the FiO2 would not have fixed this North-South syndrome. All right, we can go to the next question. And so here we're going to turn it over to Dr. Efron, I believe. Thank you, Dr. Gates. Okay, let's take a look here. Okay. A 40-year-old man presents with acute onset of shortness of breath. History is significant for recent transatlantic flight. He denies any cough, fevers, chills, or chest pain. He has not had any recent episodes of bleeding. On examination, he is in mild distress. Vital signs are a temperature of 36.9 centigrade or 98.5 Fahrenheit, heart rate of 110 beats per minute, a respiratory rate of 28 beats per minute, a blood pressure of 130 over 8 millimeters mercury, and oxygen saturation as measured by pulse oximetry as 88% on remer. Examination is significant for left calf swelling. Lungs are clear to auscultation bilaterally. Which of the following is the most appropriate step? A, parental therapeutic anticoagulation. B, CT pulmonary angiogram. C, systemic thrombolytic therapy. And D, a D-dimer test, DD. So given that, let's go to the next slide. So our rationale for this. So the patient has a high pretest probability of an acute pulmonary embolism. He has risk factors, recent air travel, symptoms, acute shortness of breath, and signs of tachycardia, tachypnea, and unilateral leg edema. That suggests treating with parenteral therapeutic anticoagulation while awaiting diagnostic test results. Anticoagulation should be initiated as soon as possible to prevent clot propagation. It has been shown to reduce mortality in patients with PE. In patients with a high clinical suspicion of PE, consensus guidelines suggest beginning treatment with parenteral anticoagulants while awaiting diagnostic test results in patients who are not at high risk of bleeding. In the absence of contraindications, CT pulmonary angiography should be performed to confirm the diagnosis of PE. However, systemic anticoagulation should start before CT. Systemic thrombolytic therapy is reserved for patients who are hypotensive, systolic blood pressure less than 90 millimeters. In hemodynamically stable patients, such as this one, the risk of bleeding outweigh the benefits of clot lysis. D-dimer testing is reserved for patients with a low probability of pulmonary embolism. Let us go to the next slide. Okay, so here are your references. You can see. Okay, next question. Thirty-nine-year-old woman who has been bed-bound presents with tachypnea and shortness of breath. The patient is intubated and point-of-care ultrasound shows massively dilated right ventricle with decreased right ventricular function. The left ventricle is hyperdynamic but has normal function. There is bowing of the ventricle septum into the left ventricle. The patient's blood pressure continues to drop and the emergency department physician is very concerned that the patient is about to go into cardiac arrest. Which of the following is the next best step in management? So, you want to sort of look at the solutions carefully. Administer Alteplase 100 milligrams over two hours and start unfractionated heparin infusion at 18 units per kilogram after the Alteplase infusion has completed. B, administer Tenecteplase 40 milligrams IV push over five seconds and start unfractionated heparin infusion also at 18 units per kilogram. Start unfractionate C, start unfractionate heparin at 18 units per kilogram and send the patient for CT angiogram of the chest. And then D, send the patient for CT angiogram of the chest prior to starting antithrombolytic therapy. So, again, you want to, they've given you several options that all may seem reasonable, but you want to pick the best option for this patient that appears to be going into shock. Next slide, please. So, the patient's presentation is most consistent with massive pulmonary embolism. Pediatric ultrasound is highly suggestive of acute right ventricular dysfunction. Given the patient's deteriorating hemodynamics and concern for impending cardiac arrest, the correct answer is Tenecteplase 40 milligrams IV push over five seconds. An alternative would be Alteplase 50 milligrams given over two minutes. If the patient were not showing signs of impending cardiac arrest, then giving Alteplase over two hours be the correct answer, but that's why it's incorrect in this situation. Given the patient's instability, choices C and D are not appropriate. Okay. Next slide. So, again, here are your references. And I think Dr. Conner is now going to take us home. Yeah. Hi. Thank you so much for the invitation and the opportunity to join this. Let's cover some nephrology questions here at the end. At least one of these will generate at least a little bit of some additional conversation beyond what was originally written here. So, the first question is a 22-year-old man undergoes resection of all but 100 centimeters of his small bowel because of mesenteric ischemia. TPN is eventually discontinued and enteral nutrition is initiated. He eventually becomes confused and has slurred speech, but is normal glycemic. The initial workup reveals an anion gap metabolic acidosis, and which of the following is the most likely cause of this patient's acidosis? A. Ketoacidosis. B. D. Lactate. C. Pyroglutamic acid. D. Selenium deficiencies. So, in this question, the correct answer is D. Lactate. We can move to the explanation. So, as people remember, if you have resection of a large portion of your small bowel, you can get small bowel syndrome and develop D. Lactate metabolic acidosis. And this is due to the fact that you get a higher amount of glycemic load delivered to the colon, and the colon bacteria then metabolize that into D. Lactate. And you absorb the D. Lactate, you have symptoms of alcohol intoxication, development of anion gap metabolic acidosis. And it's just important to remember that this is not identified with conventional L. Lactate assays. Just going back to the question for a second, on the previous slide, I think it's important to point out that, you know, selenium deficiency is not really associated with anion gap metabolic acidosis. And pyroglutamic acid, it would be awful rare in this situation. I think just clinically, this is probably also an important point to make is that you probably want to check an ethanol level in this patient as well, because there is the rare auto brewery syndrome, which can occur in some patients who have overgrowth of certain bacteria, and they auto ferment glucose into alcohol, and they could have a true alcohol related key from like an ethanol ingestion related anion gap metabolic acidosis as well. But that's not a choice here, but just something to bear in mind clinically. Let's keep moving forward to our next question. These are the references here, no reason to update those. Okay, the next question is an 18 year old female with a history of depression is admitted to the ICU after an apparent suicide attempt when she ingested 100 tablets of 220 milligram naproxens. She's intubated, has very limited neurologic exam. In the next 24 hours, she has had approximately 12 mls per kilo per hour of urine, and her serum laboratory results are a sodium of 150, potassium 3.1, chloride 104, bicarbonate 16, BUN 45, creatinine 1.8, glucose of 126, phosphorus 4.5, pH of 7.22, and iCal of 1.38, and her urine analysis reveals a pH of 7.5, a urine sodium of 30, and a urine creatinine of 72. So which of the following best explains this patient's metabolic acidosis? A, renal tubular acidosis type 1, B, RTA type 2, C, polyuric acute kidney injury, and D, pre-renal azotemia. I'll give you a second to think about that. Choice A is the correct answer, renal tubular acidosis type 1. If you want to move forward, just remember that NSAIDs can cause a lot of tubular injury, and with that tubular injury, you can get polyuria, and you can get acidosis due to an inability to excrete hydrogen ions in the setting of distal tubular dysfunction, and you'll have a high urinary pH and a sort of a development of a non-gap acidosis, which is what is an RTA type 1. An RTA type 2 is not really appropriate, that's a proximal tubular RTA, that's a proximal nephron dysfunction, and you're going to have a lot of bicarbonate wasting in that situation, but they usually do not necessarily get a pH above 7 with that. And then polyuria can occur and can certainly be a cause of acute kidney injury in this patient, as can the NSAIDs themselves, but not necessarily the explanation for the acidosis. I do want to point out that it is really very difficult to actually diagnose an RTA in the setting of acute kidney injury, so you'd like to sort of see how this resolves over time. This should get better quite quickly as the renal tubular function improves and the acute kidney injury resolves. We'll move on to the next question. Okay, so a 73-year-old woman without a significant past medical history comes in with a urinary tract infection and septic shock from a urinary source. She requires resuscitation and administration of norepinephrine infusions. In order to prevent acute kidney injury, which of the following interventions is most indicated? A, normalize her mixed venous O2 set. B, continue to resuscitate the patient with lactated ringer solution. C, titrate vasopressors to a map greater than 90. D, add inotropes to achieve a cardiac index of 4.5. Or E, place a pulmonary artery catheter to guide your hemodynamic goals. Give you a second to think about this. So I think as you read through this, there's some questions that we can clearly just eliminate here because I'm not aware that a PA catheter by itself will necessarily improve or decrease your risk for acute kidney injury, and there's really no signs here in the stem of any sort of impending cardiogenic shock. You certainly may want to evaluate to make sure that that's not the situation, but there's no real signs that this patient needs inotropes or PA catheter driven resuscitation. We have no data that you can go to the stem or to the rationale. Skipping around here a little bit, I'm going to get to the more obvious answer here in a second, but we have no data that a higher blood pressure definitively lowers AKI risks in all comers. There may be a slight lower incidence of AKI in patients with hypertension, but they told us she has no known medical history for that. So the correct answer is to continue to resuscitate with lactated ringers. Fortunately, the question didn't really ask us to resuscitate versus normal saline versus lactated ringers, at which point I think the data is a little bit all over the place on that. Most trials have shown no significant improvement in AKI or mortality rates with administration of small quantities of 0.9% sodium chloride. Some questions remain regarding large quantities of hyperchloremic solutions as the studies haven't really looked at that in terms of the randomized control trials. So the patient clearly needs ongoing volume resuscitation until they've reached certain goals, and we won't necessarily debate those goals here, but certainly to at least they reach the surviving sepsis guidelines goals of 30 mLs per kilo. Next question. All right, so 85-year-old female presents to the ICU three days after a hip replacement, has acute respiratory distress, and receives several boluses of fluids for low urine output. Normotensive, the creatinine's up to three from a baseline of 0.9, has distended neck veins, and the chest radiograph suggests acute pulmonary edema. Albumin is low. Which of the following interventions is most appropriate at this time? A, furosemide bolus alone. B, renal replacement therapy. C, furosemide infusion with boluses of albumin. D, bumetanide with IV hematolozone. We can go to the next question, to the rationale. So this rationale is quite honestly not correct. They tried to give us the answer that B, early renal replacement therapy was appropriate, but we now have many studies that support the fact that early renal replacement therapy does not necessarily improve outcome, and there's no clear other obvious indication for renal replacement therapy here, other than the fluid overload, and there has been no attempts to diuresis this patient. So generally speaking within the nephrology community, we would suggest at this point you should use high-dose diuretic therapy first, and only if that is failing and the patient has failed a furosemide stress test or high-dose diuretic therapy, then early initiation of acute renal replacement therapy may be indicated in that situation. Next question. All right, a 26-year-old, I'm just trying to remember, I think this is our last one, right? A 26-year-old man with no known history of, or history of IV drug use, has gotten unresponsive, is not known for how long they've been unconscious. I'm going to skip to the point of this, which is that the patient has an elevated creatinine and an elevated CPK, and has signs of blood positive myoglobin on the dipstick, but no blood on the urine microscopy. So how is the next, how should we manage this patient's acute kidney injury and their rhabdo? In the interest of time, the correct answer is D, and if we move forward to the rationale, we don't have any clear data that alkalizing urine is really necessary in this situation. What is necessary, if possible, is to force the urine output to go up. So I would use isotonic crystalloids to try to drive urine, and this is one of the few situations where giving diuretics and volume can be very useful, because we do want to try to drive urine output as much as possible. The creatinine is up quite a bit here, so the horse is sort of out of the barn. It may not be possible to reverse this, but it's certainly worth an effort in the short term. So I hope that's, I think that's the last of my questions, and I appreciate the opportunity, and I'll throw it back to the, to Dr. Pastoriz. Thank you, Dr. Connor. Thank you to our faculty and to the audience for attending. Again, this recording will be available in 24 to 48 hours in the content section of the course. Please be sure to join us for part two tomorrow, Tuesday, September 28, from 9 to 10 a.m. Central Time, where we will cover the topics of endocrinology, obstetrical critical care, GI liver, hematology, oncology, and surgery topics. That concludes our presentation today. Thank you so much for participating.
Video Summary
In today's Q&A session, various topics including respiratory cardiovascular disease, hemodynamics, nephrology, and ICU management were discussed. The panel of experts answered questions and provided explanations for each answer choice. Some key points from the session include the importance of early revascularization in complications of myocardial infarction, the use of beta blockers and vasodilators in descending aortic dissection, the administration of high flow nasal cannula in patients with acute hypoxic respiratory failure, the diagnosis and management of renal tubular acidosis type 1, and the treatment of acute pulmonary embolism with thrombolytic therapy when patients are hemodynamically unstable. Overall, the session provided valuable insights into the management of various medical conditions in critical care settings.
Asset Caption
Live session occurred: Monday, September 27 at 9 AM Central Time
Keywords
respiratory cardiovascular disease
hemodynamics
nephrology
ICU management
early revascularization
beta blockers
vasodilators
high flow nasal cannula
renal tubular acidosis type 1
acute pulmonary embolism
Society of Critical Care Medicine
500 Midway Drive
Mount Prospect,
IL 60056 USA
Phone: +1 847 827-6888
Fax: +1 847 439-7226
Email:
support@sccm.org
Contact Us
About SCCM
Newsroom
Advertising & Sponsorship
DONATE
MySCCM
LearnICU
Patients & Families
Surviving Sepsis Campaign
Critical Care Societies Collaborative
GET OUR NEWSLETTER
© Society of Critical Care Medicine. All rights reserved. |
Privacy Statement
|
Terms & Conditions
The Society of Critical Care Medicine, SCCM, and Critical Care Congress are registered trademarks of the Society of Critical Care Medicine.
×
Please select your language
1
English